엡실론 델타 이차 함수 - ebsillon delta icha hamsu

* 해석학 수준이 아닌 미적분학 수준에서 내는 입델 문제 위주로 다룹니다.

연세대 편입수학 1번문제는 항상 입실론델타로 나올만큼 고정문제입니다. 그렇기 때문에 합격을 생각하고 있다면 사실 입델을 절대 포기해선 안됩니다. 

지금부터 쓰는 입델 시리즈를 잘 보고  기계처럼 푸시길 바랍니다. 어차피 편수에 나오는 입델은 기계처럼 푸는법만 알아도 됩니다. 수학과 학생이라면 절대 그래선 안되지만요.

▶ 입실론 델타 

- 우리가 고등학교 시간때 배우는 '함수의 극한' , '함수의 연속'은 정의가 이러했습니다.

엡실론 델타 이차 함수 - ebsillon delta icha hamsu

다만 이것은 대학수학을 배우게되면 애매한 표현이라고 교수님들이 언급하시긴 합니다. 가까이 다가간다라는 말 때문인데 x와 a의 거리가 얼마나 가까워야 위 극한을 정의할 수 있을지 보다 '객관적인 지표'가 필요했다 이겁니다. 

그래서 등장한게 입실론 델타 논법입니다. 임의의 델타를 던져줄테니  요거가지고 알아서 잘 조립해봐 ㅎㅎ 이런 말입니다. 그러면 먼저 대학수학에서는 함수의 극한을 어떻게 정의하는지 봅시다.

엡실론 델타 이차 함수 - ebsillon delta icha hamsu

위 정의를 보시면 대충 감이 오는게 있습니다. 좌표평면 상에서 델타는 x축에서  a만큼의 위아래로 크기를 가지는 범위를 정한다는 말일 것이구요. 입실론은 y축에서 극한값 L만큼의 위아래로 크기를 가지는 범위를 정한다는 말일 것입니다. 

이를 그래프로 나타내서 보면 이러합니다.

엡실론 델타 이차 함수 - ebsillon delta icha hamsu

바로 이것이죠. 여러분들이 정상적인 고교과정을 뗐다면 집합과 명제 단원을 배우신 적이 있을겁니다. 

입실론 델타는 바로 이 집합과 명제 단원을 바탕으로 설명합니다. 델타를 집합 A라고 하죠. 그리고 입실론을 집합 B라고 한다면 집합 A는 집합 B의 부분집합으로 받아들이면 된다는 것입니다.  따라서 집합 A는 집합 B와 같거나, 작은 범위를 가져야 합니다. 우리는 이거를 그대로 델타를 입실론과 같거나 혹은 그보다 작은 범위를 가지는 경우로 만들어주면 됩니다.  

일단 가장 간단한 예제부터 하나 찬찬히 보고 다음 포스팅으로 넘어가겠습니다.

엡실론 델타 이차 함수 - ebsillon delta icha hamsu

1. 함수의 극한 (Limits of functions)

2. 극한의 엄밀한 정의, 엡실론 델타 논법(Epsilon-delta argument)

3. 극한법칙과 압축정리 (Limit laws and Squeeze Theorem)

4. 함수의 연속과 중간값 정리 (Continuity and Intermediate Value Theorem)

와 관련된 연습문제들을 모아놓은 포스트이다.

가급적 위 포스트들을 모두 공부한 후 풀어보기를 권장한다.

쉽게 풀이를 찾을 수 있는 기본 연습문제는 조금만 싣고 생각을 조금 해보아야 하는 문제들을 담았다. 

초반 문제들은 부연설명을 자세하게 달았지만 뒤로 갈수록 핵심적인 내용 외의 설명은 생략했으므로 

만약 이런 문제들을 처음 접한다면 초반 문제부터 순서대로 푸는것이 좋다.

(다만 난이도가 꼭 오름차순 배치인것은 아니다)

문제들이 계속 추가될 수도 있다.


1. 다음을 \( \epsilon-\delta \) 논법으로 증명하시오

$$ \lim_{x \to 3} 3x - 2 = 7 $$

풀이

더보기

임의의 \(\epsilon > 0\) 에 대해 \( \delta > 0 \)이 존재하여

\( 0 < |x-3| < \delta \Longrightarrow |(3x-2) - 7| < \epsilon \)을 만족하는지 살펴보아야 한다.

\( |(3x-2) - 7| = |3x - 9| = 3|x-3| < 3\times \delta \) 이므로

\(3\delta = \epsilon\), 즉 \( \delta = \dfrac{\epsilon}{3} \)으로 설정하면

\( 0 < |x-3| < \delta \Longrightarrow |(3x-2) - 7| = 3|x-3| < 3 \times \delta = 3 \times \dfrac{\epsilon}{3} = \epsilon \) 이다.

따라서 극한의 정의에 의해 \(\lim\limits_{x \to 3} 3x - 2 = 7\)이다.


2. 다음을 \( \epsilon-\delta \) 논법으로 증명하시오 (스튜어트 예제, 연세대학교 편입 2010 기출)

$$ \lim_{x \to 0} \dfrac{1}{x^{2}} = \infty $$

풀이

더보기

임의의 큰 수 \(M > 0\) 에 대해

\( 0< |x-0| < \) \(\delta\) 이면 \(\dfrac{1}{x^{2}} > M \) 을 만족하게 하는 \(\delta > 0 \) 이 존재하는지 살펴보아야 한다.

그리고 다음이 성립한다.

\( \begin{align} \dfrac{1}{x^{2}} > M &\Longleftrightarrow x^{2} < \dfrac{1}{M} \\ &\Longleftrightarrow \sqrt{x^{2}} < \sqrt{\dfrac{1}{M}} \\ &\Longleftrightarrow |x| < \dfrac{1}{\sqrt{M}} \end{align} \)

참고로 여기서 양방향 화살표를 이용했다. 즉 화살표 좌우가 필요충분조건으로 언제든지 넘나들 수 있는 논리라는 얘기다.

이제 \( \delta = \dfrac{1}{\sqrt{M}} \) 라고 한다면 \(0 < |x| < \dfrac{1}{\sqrt{M}} \)이고

위의 양방향 화살표를 타고 올라가면 원하는 부등식인 \(\dfrac{1}{x^{2}} > M \) 를 얻는다.

정리하자면, \( \delta = \dfrac{1}{\sqrt{M}} \) 으로 설정하면 

\( 0< |x-0| < \) \(\delta\) 일 때 \(\dfrac{1}{x^{2}} > M \) 을 만족하므로

무한극한의 정의에 의해 \( \lim\limits_{x \to 0} \dfrac{1}{x^{2}} = \infty \) 이다.


3. 다음을 \( \epsilon-\delta \) 논법으로 증명하시오

$$ \lim_{x \to 2} x^{2} = 4 $$

풀이

더보기

임의의 \(\epsilon > 0\) 에 대해 \( \delta > 0 \)이 존재하여

\( 0 < |x-2| < \delta \Longrightarrow |x^{2} - 4| < \epsilon \)을 만족하는지 살펴보아야 한다.

\( |x^{2} - 4| = |(x+2)(x-2)| = |x+2||x-2| \)이고,

\(|x-2|\)는 \(\delta\)보다 작으므로 \( |x+2| \) 가 어떠한 값보다 작게 만드는 방법을 찾아야 한다.

우리는 \(x=2\) 근처의 값에 관심이 있다. 따라서 \(\delta = 1\)로 설정하여

\(x\)와 \(2\)의 거리인 \( |x-2| \) 를 1보다 작게 제한시켜놓고 살펴보자.

\( \delta = 1 \) 이면 ( = \(x\)와 \(2\)의 거리를 1 미만으로 설정하면)

\( 0 < |x-2| < 1 \Longrightarrow 1<x<3, x \neq 2 \) 를 얻고

각 항에 \(2\)를 더하면 $ 3 < x+2 < 5, x + 2 \neq 4 $ 즉 $ |x+2| < 5 $를 얻는다.

또 한 편 \(\delta = \dfrac{\epsilon}{5} \) 으로 설정하면 ( = \(x\)와 \(2\)의 거리를 \(\dfrac{\epsilon}{5}\) 미만으로 설정하면) 

\(0 < |x-2| < \delta = \dfrac{\epsilon}{5} \) 이므로 \(|x-2| < \dfrac{\epsilon}{5}\) 을 얻는다.

이제 \(\delta = \text{min(}1, \dfrac{\epsilon}{5} \text{)} \) 라고 하면

\(|x+2| < 5\) 와 \(|x-2| < \dfrac{\epsilon}{5}\)를 동시에 만족하므로

\( 0 < |x-2| < \delta \Longrightarrow |x^{2} - 4| = |x+2||x-2| < 5 \times \dfrac{\epsilon}{5} = \epsilon \) 이다.

따라서 극한의 정의에 의해 \(\lim\limits_{x \to 2} x^{2} = 4\) 이다.


4. 다음을 \( \epsilon-\delta \) 논법으로 증명하시오 (고려대학교 편입 비수학과 2018 & 연세대학교 편입 2007 기출문제)

$$ \lim_{x \to 0} \dfrac{x+1}{x^{2} + 1} = 1 $$

풀이

더보기

임의의 \(\epsilon > 0\) 에 대해 \( \delta > 0 \)이 존재하여

\( 0 < |x| < \delta \Longrightarrow \left|\dfrac{x+1}{x^{2} + 1} - 1\right| < \epsilon \)을 만족하는지 살펴보아야 한다.

\( \left|\dfrac{x+1}{x^{2} + 1} - 1\right| = \left|\dfrac{x-x^{2}}{x^{2} + 1}\right| = \dfrac{|x-1|}{|x^{2}+1|}|x| \) 이고

\(\delta = 2\) 라고 하면 \(0 < |x| < 2 \Longrightarrow -2 < x < 2, x \neq 0\) 이고 각 항에 \(-1\)을 더하면

\( -3< x-1 < 1, x \neq 1 \)을 얻는다.

따라서 \(\delta = 2\) 일 때 \(|x-1| < 3\) 이다. (왜 $1$이 아닌지는 직접 대입해보라.)

또 역시 \(\delta = 2\)에서는 \( 0 < |x| < 2 \) 이므로 
\( 0< x^{2} < 4 \Longrightarrow 1< |x^{2}+1| < 5\) 이고 \( \dfrac{1}{5}<\dfrac{1}{|x^2 + 1|} < 1 \) 을 얻는다.

그리고 \( \delta = \dfrac{\epsilon}{3} \) 이라 하면 \( |x| < \dfrac{\epsilon}{3} \)을 얻으므로

\(\delta = \text{min(}2, \dfrac{\epsilon}{3} \text{)} \) 이라고 설정하면

\(|x-1| < 3\), \( \dfrac{1}{|x^2 + 1|} < 1 \), \( |x| < \dfrac{\epsilon}{3} \) 을 모두 만족시키므로

\( 0 < |x| < \delta \Longrightarrow \left|\dfrac{x+1}{x^{2} + 1} - 1\right| = \\  \dfrac{|x-1|}{|x^{2}+1|}|x| < 3 \times 1 \times \dfrac{\epsilon}{3} = \epsilon\) 이다.

따라서 극한의 정의에 의해 \( \lim\limits_{x \to 0} \dfrac{x+1}{x^{2} + 1} = 1 \) 이다.


5. 다음을 \( \epsilon-\delta \) 논법으로 증명하시오 (연세대학교 편입 2012 기출문제)

$$ \lim_{x \to 2} \dfrac{1}{ \sqrt{x} + \sqrt{2} } = \dfrac{1}{2\sqrt{2}}$$

풀이

더보기

임의의 \(\epsilon > 0\) 에 대해 \( \delta > 0 \)이 존재하여

\( 0 < |x-2| < \delta \Longrightarrow \left| \dfrac{1}{ \sqrt{x} + \sqrt{2} } - \dfrac{1}{2\sqrt{2}} \right| < \epsilon \)을 만족하는지 살펴보아야 한다.

한편 다음이 성립하고

\( \left| \dfrac{1}{ \sqrt{x} + \sqrt{2} } - \dfrac{1}{2\sqrt{2}} \right| = \left| \dfrac{ \sqrt{2} - \sqrt{x} }{ 2\sqrt{2} (\sqrt{x} + \sqrt{2} ) } \right| = \dfrac{|x-2|}{2\sqrt{2}(\sqrt{x} + \sqrt{2})^{2}} \)

\(|x-2|\)는 \(\epsilon\) 을 포함한 식으로 원하는 값보다 작게 쉽게 만들 수 있으므로

\( \dfrac{1}{(\sqrt{x} + \sqrt{2})^{2} } \) 를 어떤 값보다 작게 만들어보자.

\(|x-2| < a\)로 범위를 제한했다 치면

\( \begin{align} 2-a<x \Longrightarrow &\sqrt{2-a} < \sqrt{x} \\ \Longrightarrow &\sqrt{2-a} + \sqrt{2} < \sqrt{x} + \sqrt{2}  \end{align} \)

계산을 간편하게 하기 위해 \(\sqrt{2-a}\) 를 \(k\sqrt{2}\), (\( k \in \mathbb{R} \)) 꼴로 만들어보자

\(a = \dfrac{3}{2}\) 이면 \(\sqrt{2-a} = \sqrt{\dfrac{1}{2}} = \dfrac{\sqrt{2}}{2}\) 이므로 \( \sqrt{2} + \sqrt{2-a} = \dfrac{3}{2}\sqrt{2} \) 이다.

따라서 \(a = \dfrac{3}{2} \) 이면

\( \begin{align} \dfrac{3}{2}\sqrt{2} < \sqrt{x} + \sqrt{2} \Longrightarrow &\dfrac{9}{2} < (\sqrt{x} + \sqrt{2})^{2} \\ \Longrightarrow &\dfrac{1}{ (\sqrt{x} + \sqrt{2})^{2} } < \dfrac{2}{9} \end{align} \)

아까 위에서 \(|x-2|\)는 원하는 값보다 쉽게 작게 만들 수 있다고 하였다.

그리고 위에서 \(|x-2| < a = \dfrac{3}{2} \) 인 상황에서는 \( \dfrac{1}{ ( \sqrt{x} + \sqrt{2} )^{2} } < \dfrac{2}{9} \)라고 하였으므로

 원하는 값 \(\epsilon \times 2\sqrt{2} \times \dfrac{9}{2} = 9\sqrt{2}\epsilon \) 으로 설정하면 

\(|x-2| < \dfrac{3}{2} \) \( |x-2| <9\sqrt{2}\epsilon \)  동시에 만족하는 상황에서

\( \dfrac{|x-2|}{2\sqrt{2}(\sqrt{x} + \sqrt{2})^{2}} < \dfrac{1}{2\sqrt{2}} \times \dfrac{2}{9} \times 9\sqrt{2}\epsilon = \epsilon \) 을 만족하여 원하는 꼴이 유도될 것이다.

따라서 \(\delta = \text{min(}\dfrac{3}{2}, 9\sqrt{2}\epsilon \text{)} \) 라고 하면

\( 0 < |x-2| < \delta \) 일 때 \( \left| \dfrac{1}{ \sqrt{x} + \sqrt{2} } - \dfrac{1}{2\sqrt{2}} \right| = \dfrac{|x-2|}{2\sqrt{2}(\sqrt{x} + \sqrt{2})^{2}} < \epsilon \) 을 만족하므로

극한의 정의에 의해 \( \lim\limits_{x \to 2} \dfrac{1}{ \sqrt{x} + \sqrt{2} } = \dfrac{1}{2\sqrt{2}} \)이다.


6. 다음을 \( \epsilon-\delta \) 논법으로 증명하시오 (연세대학교 편입 2006 기출문제)

$$ \lim_{x \to 0} x \cos{\left( \dfrac{1}{x} \right)} = 0 $$

풀이

더보기

임의의 \(\epsilon > 0\) 에 대해 \( \delta > 0 \)이 존재하여

\( 0 < |x| < \delta \Longrightarrow \left| x \cos{ \left( \dfrac{1}{x} \right) } \right| < \epsilon \)을 만족하는지 살펴보아야 한다.

\( \left| x \cos{ \left( \dfrac{1}{x} \right) } \right| = |x| \left| \cos{\left( \dfrac{1}{x} \right)} \right| \) 이고

모든 \(x\) 에 대해 \( \cos{ \left( \dfrac{1}{x} \right) } \le 1\) 이므로 

\(\delta = \epsilon\) 이라고 하면

\( 0 < |x| < \delta \Longrightarrow \left| x \cos{ \left( \dfrac{1}{x} \right) } \right| = |x| \left| \cos{ \left( \dfrac{1}{x} \right) } \right| < 1 \times \epsilon = \epsilon \) 이다.

따라서 극한의 정의에 의해 \( \lim\limits_{x \to 0} x \cos{ \left( \dfrac{1}{x} \right) } = 0 \)


7. 다음을 \( \epsilon-\delta \) 논법으로 증명하시오 (고려대학교 편입 수학과 2018 기출문제)

$$ \lim_{x \to 0} (1 + \text{sin}x)^{2} = 1 $$

풀이

더보기

먼저 \( |x| < \dfrac{\pi}{2} \) 이면 \( |\text{sin} x| \le |x| \) 임을 증명하자.

\(x = 0\) 이면 \(0 \le 0\) 이므로 성립한다.

\(0 < x < \dfrac{\pi}{2} \) 인 경우

엡실론 델타 이차 함수 - ebsillon delta icha hamsu

왼쪽 그림은 점 \(O\)를 중심으로 반지름이 1인 원에
원 위의 두 점 \(A, B\)와 점 \(O\)를 꼭짓점으로 하는 삼각형이다.

삼각형의 넓이는 \(1 \times 1\text{sin}x \times \dfrac{1}{2} = \dfrac{\text{sin}x}{2}\) 이고

부채꼴 \(OAB\)의 넓이는 \( \dfrac{1}{2} \times 1^{2} \times x = \dfrac{x}{2} \) 이다.

부채꼴의 넓이가 삼각형보다 더 크거나 같으므로

\(\dfrac{\text{sin}x}{2} \le \dfrac{x}{2} \Longrightarrow \text{sin}x \le x \)

\(-\dfrac{\pi}{2} < x < 0 \) 인 경우

\(-x = t\) 라고 하면 \(0 < t < \dfrac{\pi}{2} \) 이고

이 범위에서는 \(\text{sin}t \le t \) 임을 위에서 보였다.

따라서 \( \text{sin}t \le t \) 의 \(t\)에 \(-x\)를 대입하면 \( \text{sin}(-x) \le -x \) \(\Longrightarrow \) \( -\text{sin}x \le -x \)

따라서 \( |x| < \dfrac{\pi}{2} \) 이면 \( |\text{sin}x| < |x| \) 이다.

이제 본론으로 돌아오자.

임의의 \(\epsilon > 0\) 에 대해 \( \delta > 0 \)이 존재하여

\( 0 < |x| < \delta \Longrightarrow |(1 + \text{sin}x)^{2} -1| < \epsilon \)을 만족하는지 살펴보아야 한다.

\( |(1 + \text{sin}x)^{2} -1| = |\text{sin}x||\text{sin}x + 2| \)

\(|x| < \dfrac{\pi}{2} \) 이면 \( |\text{sin}x| < |x| \)임은 위에서 보였고 

여기서 \(|x| < \dfrac{\epsilon}{3} \)을 추가로 만족한다면

\( |\text{sin}x| < |x| < \dfrac{\epsilon}{3}\) 를 만족한다.

그리고 \(|\text{sin}x + 2| \le 3\)이므로 \(\delta = \text{min(}\dfrac{\pi}{2}, \dfrac{\epsilon}{3} \text{)} \) 라고 하면

\( 0 < |x| < \delta \Longrightarrow |(1 + \text{sin}x)^{2} -1| = |\text{sin}x||\text{sin}x + 2| < \dfrac{\epsilon}{3} \cdot 3 = \epsilon \)

이므로 극한의 정의에 의해 \( \lim\limits_{x \to 0} (1 + \text{sin}x)^{2} = 1 \)


8. 다음을 \( \epsilon-\delta \) 논법으로 증명하시오 (연세대학교 편입 2009 기출문제)

$$ \lim_{x \to 2} \left(x^{3} - 3x + 1\right) = 3 $$

풀이

더보기

임의의 \(\epsilon > 0\) 에 대해 \( \delta > 0 \)이 존재하여

\( 0 < |x-2| < \delta \Longrightarrow \left| x^{3} - 3x + 1 - 3 \right| < \epsilon \)을 만족하는지 살펴보아야 한다.

\( \left| x^{3} - 3x + 1 - 3 \right| = (x+1)^{2}|x-2| \) 이고

\(\delta = 1\) 이라 하면 \( 0 < |x-2| < 1 \Longrightarrow 2<x+1<4, x \neq 3 \) 이므로

\(|x+1|<4 \Longrightarrow (x+1)^{2} < 16 \) 를 얻는다.

따라서 \(\delta = \text{min(}1, \dfrac{\epsilon}{16} \text{)} \) 이라고 하면

\( 0 < |x-2| < \delta \Longrightarrow \left| x^{3} - 3x + 1 - 3 \right| = (x+1)^{2}|x-2| < 16 \cdot \dfrac{\epsilon}{16} = \epsilon \) 이다.

따라서 극한의 정의에 의해 \(\lim\limits_{x \to 2} \left(x^{3} - 3x + 1\right) = 3\) 이다.


9. 함수 \(f\) 가 \(x=3\)에서 연속인지 불연속인지 \(\epsilon - \delta\) 논법을 사용하여 증명하시오.

(단 \( \mathbb{Q} \)는 유리수 전체의 집합이다.) (연세대학교 편입 2015 기출문제)

$$ f(x) = \begin{cases} \begin{align} x^{2}& &(x \in \mathbb{Q}) \\ 3x& &(x \notin \mathbb{Q}) \end{align} \end{cases}$$

풀이

더보기

연속의 정의에 의해 \(\lim\limits_{x \to 3} f(x) = f(3) = 9\) 임을 증명하면 \(x=3\) 에서 연속임이 증명된다.

\(x \in \mathbb{Q}\) 일 때 \(f(x) = x^{2}\) 이고 \(x \notin \mathbb{Q}\) 일 때 \(f(x) = 3x\) 이므로

임의의 \(\epsilon > 0\)에 대해 \(\delta_{1}, \delta_{2} > 0\)이 존재하여

$$ \begin{align} & 0 < |x-3| < \delta_{1} \Longrightarrow |x^{2} - 9| < \epsilon \\ & 0<|x-3|<\delta_{2} \Longrightarrow |3x - 9| < \epsilon \end{align} $$

을 만족함을 보이자.

\(x \in \mathbb{Q}\) 인 경우

\(|x^{2}-9| =|x+3||x-3| \) 이고 \(0 < |x-3| < 1 \) 로 제한하면

\(0 < |x-3| < 1 \Longrightarrow -1< x-3 < 1, x \neq 3 \Longrightarrow 5 < x+3 < 7, x \neq 6 \)

따라서 \( |x+3| < 7 \) 를 얻는다.

즉 \( 0 < |x-3| < 1 \) 이면 \(|x+3| < 7\)이다.

또 \( 0 < |x-3| < \dfrac{\epsilon}{7} \) 으로 제한하면 

\( 0 < |x-3| < \dfrac{\epsilon}{7} \)일 때  \( |x-3| < \dfrac{\epsilon}{7} \) 이므로

\(\delta_{1} = \text{min(}1, \dfrac{\epsilon}{7} \text{)} \) 으로 설정하면

\(0 < |x-3| < \delta_{1} \Longrightarrow |x^{2} - 9| = |x+3||x-3| < 7 \cdot \dfrac{\epsilon}{7} = \epsilon \)

\(\delta_{1} > 0\)의 존재성을 보였으므로 \(x \in \mathbb{Q} \) 에 대해서는 극한이 존재한다.

\(x \notin \mathbb{Q}\) 인 경우

\(|3x - 9| = 3|x-3|\) 이므로 \(\delta_{2} = \dfrac{\epsilon}{3} \) 으로 설정하면

\( 0 < |x-3| < \delta_{2} \Longrightarrow |3x-9| = 3|x-3| < 3 \cdot \dfrac{\epsilon}{3} = \epsilon \)

\(\delta_{2} > 0\)의 존재성을 보였으므로 \(x \notin \mathbb{Q} \) 에 대해서도 극한이 존재한다.

이제 \( \delta = \text{min(}\delta_{1}, \delta_{2}\text{)} \) 라고 하면

\(x \in \mathbb{R} \) 에서 \(0 < |x-3| < \delta \Longrightarrow |f(x) - 9| < \epsilon\) 가 성립하므로

극한의 정의에 의해 \(\lim\limits_{x \to 3} f(x) = 9\) 이다. 따라서 \(f(x)\) 는 \(x=3\) 에서 연속이다.


10. 함수 \(f\)가 다음과 같을 때 어떤 점에서 연속인지 설명하시오. (스튜어트 연습문제, 연세대학교 편입 2010 기출)

$$ f(x) =  \begin{cases} \begin{align} 0 &&(x \in \mathbb{Q}) \\ 1 &&(x \notin \mathbb{Q}) \end{align} \end{cases}$$

풀이

더보기

\(f(x)\) 가 임의의 점 \(x = a\)에서 연속이려면

\(\lim\limits_{x \to a} f(x) = f(a) \) 를 만족해야 한다. 

연속의 조건 중 "극한이 존재한다"가 모순임을 보임으로써 연속이 아님을 증명할 것이다.

\(\lim\limits_{x \to a} f(x) \) 가 \(L\)로 존재한다고 가정하여 모순을 이끌어내자 (귀류법)

\( \lim\limits_{x \to a} f(x) = L \) 이므로 임의의 \(\epsilon > 0\)에 대해 \(\delta_{1}, \delta_{2} > 0\)가 존재하여

\(( x \in \mathbb{Q}) \) 에서 \(0 < |x-a| < \delta_{1} \Longrightarrow |f(x)-f(a)| < \epsilon \) 을 만족하고

\( (x \notin \mathbb{Q}) \) 에서 \(0 < |x-a| < \delta_{2} \Longrightarrow |f(x) - f(a)| < \epsilon \) 을 만족하여야 한다.

임의의 \(\epsilon > 0\)에 대해서 만족하여야 하는데, \(\epsilon = \dfrac{1}{4} \) 라고 설정해보면

\( (x \in \mathbb{Q}) \) 인 경우 \(f(x) = 0\) 이므로

\(0 < |x-a| < \delta_{1} \Longrightarrow |0-f(a)| < \dfrac{1}{4} \Longrightarrow -\dfrac{1}{4} < f(a) < \dfrac{1}{4} \) 이고

\( (x \notin \mathbb{Q}) \) 인 경우 \(f(x) = 1\) 이므로

\(0 < |x-a| < \delta_{1} \Longrightarrow |1-f(a)| < \dfrac{1}{4} \Longrightarrow \dfrac{3}{4} < f(a) < \dfrac{5}{4} \) 인데 

이를 동시에 만족하는 \(f(a)\) 는 존재할 수 없다.

따라서 가정인 \( \lim\limits_{x \to a} f(x) = L \) 는 모순이고 극한이 존재하지 않는다.

모든 점에서 극한이 존재하지 않으므로 모든 점에서 연속이 아니다.


11. 함수 \(f : \mathbb{R} \rightarrow \mathbb{R}\) 가 \(\lim\limits_{x \to 0} f(x) = 1 \)을 만족한다고 하자.

그러면 \(\lim\limits_{x \to 0} (f(x))^{2} = 1 \) 을 만족함을 \(\epsilon - \delta\) 논법으로 증명하시오 (연세대학교 편입 2021 기출문제)

풀이

더보기

임의의 \(\epsilon > 0\) 에 대해 \( \delta > 0 \)이 존재하여

\( 0 < |x| < \delta \Longrightarrow \left| (f(x))^{2} - 1 \right| < \epsilon \)을 만족하는지 살펴보아야 한다.

우선 다음을 관찰하자.

\( \left| (f(x))^{2} - 1 \right| = |f(x) + 1||f(x) - 1| \) 

\( |f(x)+1| \) 와 \( |f(x) - 1| \) 이 모두 무언가보다 작게 만들어야 한다.

후자는 \(\lim\limits_{x \to 0} f(x) = 1 \) 가 주어져 있으므로 쉽게 무언가 보다 작게 만들 수 있다. 

전자에 대해 구해보자.

\(\lim\limits_{x \to 0} f(x) = 1 \) 가 주어져 있다.

따라서 \(\delta_{1} > 0 \) 이 존재하여 \(0 < |x| < \delta_{1} \Longrightarrow |f(x) - 1| < 1 \) 을 만족한다.

그리고 삼각부등식에 의해 이러한 \(x\) 에서는 \(|f(x) + 1| = |(f(x) - 1) + 2| < |f(x) - 1| + |2| < 3\) 을 만족한다.

즉, \(0 < |x| < \delta_{1} \) 에서는 \(|f(x) + 1| < 3 \) 이다.

또 \(\delta_{2} > 0\) 역시 존재하여 \(0 < |x| < \delta_{2} \Longrightarrow |f(x) - 1| < \dfrac{\epsilon}{3} \) 을 만족하므로

\(\delta = \text{min(}\delta_{1}, \delta_{2}\text{)}\) 으로 설정하면

\( 0 < |x| < \delta \Longrightarrow \left| (f(x))^{2} - 1 \right| = |f(x) + 1||f(x) - 1| < 3\times\dfrac{\epsilon}{3} = \epsilon \) 이므로

극한의 정의에 의해 \(\lim\limits_{x \to 0} (f(x))^{2} = 1 \) 이다.


12. 함수 \(f \) 가 다음과 같이 정의되어 있다.

$$ f(x) =  \begin{cases} \begin{align} &\text{sin}x &(x \in \mathbb{Q}) \\ &x(x+1) &(x \notin \mathbb{Q}) \end{align} \end{cases} $$

\(\lim\limits_{x \to 0} \dfrac{\text{sin}x}{x} = 1 \) 임을 이용하여 \(f\) 가 \(x=0\) 에서 미분가능함을 \(\epsilon - \delta \) 논법으로 증명하시오. (\(\mathbb{Q}\) 는 유리수 전체의 집합) (연세대학교 편입 2020 기출)

(이 문제는 미분가능성에 대해 먼저 공부한 사람만 풀면 된다.)

풀이

더보기

\(x=0\) 에서 미분 가능하려면 미분계수의 정의에 따라 다음의 극한이 존재하면 된다.

$$ \lim_{x \to 0} \dfrac{f(x) - f(0)}{x - 0} $$

\(0 \in \mathbb{Q} \) 이므로 \(f(0) = 0 \) 이고 따라서 다음의 극한이 존재함을 보이면 된다.

$$ \lim_{x \to 0} \dfrac{f(x)}{x} $$

\((x \in \mathbb{Q})\) 일 때는 \(\dfrac{f(x)}{x} = \dfrac{\text{sin}x}{x} \) 이고

\(\lim\limits_{x \to 0} \dfrac{\text{sin}x}{x} = 1 \) 가 주어져 있으므로

\( \delta_{1} > 0 \) 이 존재하여

\(0 < |x| < \delta_{1} \Longrightarrow \left|\dfrac{\text{sin}x}{x} - 1 \right| < \epsilon \) 을 만족한다.

\((x \notin \mathbb{Q})\) 일 때는 \(\dfrac{f(x)}{x} = \dfrac{x(x+1)}{x} = x+1\) 이므로

\(\delta_{2} > 0 \) 가 존재하여 \(0 < |x| < \delta_{2} \Longrightarrow |x+1 - 1| < \epsilon \) 을 만족하는지 살펴보아야 한다.

\( |x+1 - 1| = |x| \) 이므로 \(\delta_{2} = \epsilon\) 이라고 하면 

\(0 < |x| < \delta_{2} \Longrightarrow |x+1 - 1| = |x| < \delta = \epsilon \) 이므로 만족한다.

따라서 \(\delta = \text{min(}\delta_{1}, \delta_{2}\text{)}\) 으로 설정하면

\(0 < |x| < \delta_{1}\), \(0 < |x| < \delta_{2}\) 가 동시에 만족하므로

\((x \in \mathbb{Q})\) 일 때와 \((x \notin \mathbb{Q})\) 일 때 모두 \( \left| \dfrac{f(x)}{x} - 1 \right| < \epsilon \) 이 만족하게 되고

따라서 \(( x \in \mathbb{R} )\) 일 때 \(0 < |x| < \delta \Longrightarrow \left| \dfrac{f(x)}{x} - 1 \right| < \epsilon \) 이 만족한다.

따라서 극한의 정의에 의해 \( \lim\limits_{x \to 0} \dfrac{f(x)}{x} = 1 \) 이고 \(f(x)\) 는 \(x=0\) 에서 미분가능하다.


13. \(f(x) = x^{2} + 10\text{sin}x\) 이면 \(f(c) = 1000 \) 을 만족하는 \(c\) 가 존재함을 보여라.

(스튜어트 연습문제)

풀이

더보기

\(f(x)\) 는 다항함수와 삼각함수의 합이므로 각각이 정의되는 정의역의 교집합에서 연속이다.

따라서 실수 전체에서 연속이다. 

\( f(a) < 1000 \) 인 \( a \) 를 찾고

\( f(b) > 1000 \) 인 \( b \) 를 찾아 중간값 정리를 이용하여

\( f(c) = 1000 \) 인 \(c \in (a, b)\)  존재함을 증명할 것이다.

\(-10 \le 10 \text{sin}x \le 10 \) 이므로

\(x = \sqrt{900}\) 이라 하면 \(f(x) = 900 + 10\text{sin}x \le 910 \) 이다.

 \(x = \sqrt{1100}\) 이라 하면 \(f(x) = 1100 + 10\text{sin}x \geq 1090 \) 이다.

따라서 중간값 정리에 의해 \( f(c) = 1000 \) 이 되는 \(c \in (\sqrt{900}, \sqrt{1100})\) 이 존재한다.


14. \(\text{cos}x = x^{3}\) 에 대해 방정식이 적어도 하나의 실근을 가짐을 증명하라. (스튜어트 연습문제)

풀이

더보기

\(f(x) = \text{cos}x - x^{3}\) 라고 하자.

삼각함수와 다항식의 차이므로 \(f(x)\)는 실수 전체에서 연속이다.

그리고

\(f(0) = 1 > 0\),

\(f(\pi) = -1 - \pi^{3} < 0\) 이므로

중간값 정리에 의해 \(f(c) = 0 \) 을 만족하는 \(c \in (0, \pi) \) 가 존재한다.

이는 방정식의 근이므로 \(\text{cos}x = x^{3}\) 는 적어도 하나의 실근을 갖는다.